Jump to content

Would this experiment be proved wrong?


abhilash

Recommended Posts

This article is an abridged version of the original article posted in google groups. For further proofs or clarifications please download the file titled electrodynamics and magnetism.pdf from the link that is given at the last part of this article.

As a preface – this experiment is based on these well established facts that

 

1) Current is the rate of flow of charge – any charge, not just the flow of electrons.

2) This flow of charge produces a magnetic field around itself/conductor.

3) Classical physics is purely deterministic and that definite causes should have definite results.

4) Observers in the same frame of reference (and who are at the same point and stationary to each other) should agree with same or similar incidents (ie, all parameters remaining the same).

Before coming to the experiment, the reasons for this experiment are explained otherwise its necessity would not be appreciated.

 

THE ANOMALY:-

Consider the setup - AB is an infinitely long current carrying conductor carrying current I. let CD be a metal strip positioned near to the conductor.

fig1h.png

Case 1) The conductor AB is at rest with respect to the metal strip CD (fig 1A)

Here there will be a magnetic field in and around the metal strip CD, but there won’t be any potential developed across it (since there is no magnetic force acting on the electrons in CD).

Case 2) the potential applied to AB is reversed (so drift velocity is negative), and CD is moving with a negative drift velocity– i.e., here essentially CD is at rest with the conducting electrons in AB and moving with respect to non conducting charges. Here, there will be a magnetic field and there will be magnetic force acting on the conducting electrons of the metallic strip CD. This causes a potential to be developed across CD.

 

Now, it can be seen that in both cases the metal strip is under exactly similar conditions as shown in Fig 2- CD is stationary with respect to one kind of charges in AB and is moving with respect to the other (opposite) kind of charge.

fig2.png

 

In case1) the metallic strip CD sees a current due to the flow of electrons in the conductor AB and in case 2) the metallic strip CD sees a current due to the (net) flow of nuclear positive charge in conductor AB.

 

In case1) the flowing electrons in AB produce a magnetic field.

In case2) it is the net nuclear positive charge that is moving. Shouldn’t they produce a magnetic field?

 

However it is well known fact that the potential developed in the two cases are different.

 

Now if one observes closely, this anomaly of measuring different potential under identical situation can be easily explained.

 

1) The magnetic field which acts on the metallic strip CD is also acting on the probes of the potentiometer (Fig 2B) which develops the same potential across it (i.e., the probes of the wire that are equi-distant from the wire AB are at equi -potential and hence the potentiometer reads zero potential. This creates a situation where the voltage developed can’t be measured directly.

2) The calculated potential developed in case 2) is generally of the order of Pico-volts (for a few amps) and this clubbed with the above fact (1) makes it even harder to detect.

fig2b.png

 

Assuming it is due to these reasons that the measured potential was different, one can straightforwardly come to the conclusion that the force (in this case and at “atomic” level) acting on the electrons in CD has to be of the form –

 

F=k q Q V^2/r (force in case 2 which has to be true in case 1 also)*

 

Where k = a constant,

q = charge of the electrons (in CD),

Q = concerned conducting Charge in AB,

V = drift velocity of electrons (relative velocity between the charges in motion).

 

(* it can be seen seen that at the “macroscopic” level this still retains the equation for net force as F= Bqv which is independent of the relative velocity of the moving charges and is described in the orginal version of this article and it's link is given at the final part of this article).

 

(It should be noted that force between charges in motion are not always related to square of their relative velocities, which is mentioned in the later part of this article**).

 

What was said above can be easily proved with the help of an experiment as described below.

 

THE EXPERIMENT:-

 

(This experiment is based on above equation that this force on the electrons is proportional to square of the relative velocity and sign of the concerned charges. So materials with different drift velocity (or with different current carriers) should exert different force on charges placed near to them).

 

Consider a straight long tube CD which on conduction electro-deposition of copper takes place at the cathode (-). The cathode is in contact with a long piece of wire AB (as shown in fig 3) which is connected to the negative terminal of the battery and the anode is connected to the positive terminal. PQRS is a piece of wire or a metal plate shaped as shown and is placed near this (AB – CD) arrangement.

fig3v.png

 

On the tube, the carriers of current are the positive ions where as in the wire; it is the electrons that conduct electricity. On conduction (as per the equation-given above), the conducting electrons in AB repel electrons in PQRS away from it whereas the moving cat ions on the tube attract electrons towards it. This causes a potential to be developed in PQRS and can be measured directly. (Here the length of QP/RS has to be much smaller than the length of AB and CD preferably QP=1cm and QR= 200cm).

 

In the above experiment, in place of AB and CD, n-type and P-type semiconductors can be used which should develop much more voltage across PS (of the order of nano-volts) since the drift velocity of electrons in semiconductors is much greater.

 

So the question is –would there be a potential developed across P-S?

 

========================================================

 

** Consider a straight long wire AB carrying current I and a charge Q is moving perpendicular to it with a velocity V as shown in the figure 4. Here as the particle moves, it can be seen that its “r”, the distance between AB and Q that varies and hence the magnitude of the magnetic field changes and thus, it is magnetic induction that plays here and is very different from the above case.

 

V = dr/dt

fig4t.png

 

Here clearly the equation for the force take the form F= BQV which is independent of the square of their relative velocities.

For a detailed description about this article please download the file titled Electrodynamics and Magnetism.pdf from the site http://groups.google.com/group/electrodynamicsandmagnetism

 

Please see this experiment as a classical one since I prefer a rational and logical answer thats in agreement with the four facts that was quoted at the beginning of this article. Additionally, I request you to see me as a person who believes in the theory of relativity and sees it with great awe and respect.

(If you wish to chat with me regarding the content of this article, please reply to me with your expected/preferred time (in GMT) so that we can share our views).

 

Many thanks in advance

Abhilash j pillai.

Edited by abhilash
Link to comment
Share on other sites

Interesting, Abhilash. I'd say yes, there should be a PQRS potential. But note that whatever I expect, the experimental results are more important, and there are some issues with your opening well-established facts:

 

1) Current is the rate of flow of charge – any charge, not just the flow of electrons.

 

Yes, a stream of copper ions carries a current, but don't forget displacement current. This is not a flow of charge.

 

2) This flow of charge produces a magnetic field around itself/conductor.

 

Yes, we observe a magnetic field around the current in the wire, but this is merely one "aspect" of the electromagnetic field. It becomes observable in this situation, but it isn't actually created. In similar vein you will detect a magnetic field if you moved past an electron. Your motion altered your observation of the electromagnetic field, it didn't create a new field.

 

3) Classical physics is purely deterministic and that definite causes should have definite results.

 

Agreed. But see Jefimenko's equations and note this: "There is a widespread interpretation of Maxwell's equations indicating that time variable electric and magnetic fields can cause each other. This is often used as part of an explanation of the formation of electromagnetic waves. However, Jefimenko's equations show otherwise. [3] Jefimenko says, "...neither Maxwell's equations nor their solutions indicate an existence of causal links between electric and magnetic fields. Therefore, we must conclude that an electromagnetic field is a dual entity always having an electric and a magnetic component simultaneously created by their common sources: time-variable electric charges and currents."

 

You might be interested in Vector potential, electromagnetic induction and ‘physical meaning’ by G Giuliani where the abstract says this:

 

"A forgotten experiment by Andre Blondel (1914) proves, as held on the basis of theoretical arguments in a previous paper, that the time variation of the magnetic flux is not the cause of the induced emf; the physical agent is instead the vector potential through the term −∂ A/∂t (when the induced circuit is at rest). The ‘good electromagnetic potentials’ are determined by the Lorenz condition and retarded formulae. Other pairs of potentials derived through appropriate gauge functions are only mathematical devices for calculating the fields; they are not physically related to the sources. The physical meaning of a theoretical term relies, primarily, on theoretical grounds; a theoretical term has a physical meaning if it cannot be withdrawn without reducing the predictive power of a theory or, in a weaker sense, if it cannot be withdrawn without reducing the descriptive proficiency of a theory."

 

Another very interesting paper is The Refractive Index in Electron Optics and the Principles of Dynamics by Ehrenberg and Siday in 1948. This classical paper predicted what is now termed the Aharonov-Bohm effect.

Link to comment
Share on other sites

THANKS a lot Farsight for your reply and especially for remarking it as “Interesting”

 

for this article was often misleadingly interpreted (by almost everyone) as something that was against the well established theory of relativity in majority of the newsgroups and hence ridiculed.

 

And I do completely agree with your comments regarding the facts 1) and 2). The purpose I added these four (incomplete) facts was because of the experience I had got from other newsgroups and believing that this should help me in expressing my viewpoint.

 

Since this article wasn’t concerned with displacement current and since I wanted this article to be as concise as possible, I didn’t give much relevance to it which I should have. (Your comment definitely fills this gap –thank you).

Similarly is the case with expressing fact 2)- I do agree with your opinion regarding this context sensitive “producing or creating” magnetic field terminology and even now I am running out of words (perhaps its because its late night two now).

 

And thankyou for referring me to Jefimenko's equations and the abstract is really interesting since it’s my pleasure to be with these kinds of stuff especially regarding induction on which I wish to write an article.

Thank you.

Link to comment
Share on other sites

It's not a measurement artifact. What you have missed is that things won't look the same in the frame of the moving metal strip in case #2. You have to consider length contraction of the wire, which gives you a net charge density, and thus an electric field. In the metal strip's frame, this will be cause a potential difference.

 

http://physics.weber.edu/schroeder/mrr/MRRtalk.html

Link to comment
Share on other sites

A pleasure, abhilash. When it comes to relativity, do bear in mind Minkowski’s Space and Time , and look at what he said about two pages from the end:

 

"Then in the description of the field produced by the electron we see that the separation of the field into electric and magnetic force is a relative one with regard to the underlying time axis; the most perspicious way of describing the two forces together is on a certain analogy with the wrench in mechanics, though the analogy is not complete".

 

This is echoed in Maxwell's On Physical Lines of Force where he said:

 

"A motion of translation along an axis cannot produce a rotation about that axis unless it meets with some special mechanism, like that of a screw."

 

By the way, is the current going the wrong way in wire-section AB in your figure 3? I'm not sure if this would change the outcome. Perhaps it would be simpler if you replaced wire-section AB with an electron beam? Meanwhile do read up on the Ehrenberg and Siday paper and the Aharanov-Bohm effect wherein "the electromagnetic potential A is seen as being more fundamental or "real"; the E and B fields can be derived from the potential A, but the potential can not be derived from the E and B fields."

Link to comment
Share on other sites

Ok and thank you swansant for replying to me.

 

Let me modify case 1 and case 2 as two different (sets) in a single system.

 

Here CD is in between AB and XY as shown in the fig-a. AB is stationary with respect to CD and a current is flowing in AB as shown.

CD is at rest with respect to the non-conducting charges in AB and the conducting electrons in AB are moving with a positive velocity Vd.

 

figcase1and2withqinbetw.png

 

However with XY, the potential across XY is reversed to that of AB causing the electrons in XY to move with a negative drift velocity, and at that very instant,

XY starts to move with a positive drift velocity.

 

(please see it this way, at the very instant the electrons in XY starts to move with an incrementally small negative velocity, XY also starts to move with an equally but positive incremental velocity). So essentially, CD is at rest with respect to conducting electrons in XY while it is moving with respect to non conducting charges in XY).

So in this frame with respect to CD, exactly identical conditions exist, CD is at rest with respect to one kind of charge and is in motion with respect to the opposite kind of (net) charge and there is just one frame of reference which is at rest with respect to Q. But in set {AB-CD} system*, there is NO force where as in set {XY-CD} system there is. This is the anomaly, two different results for a similar or same cause.

 

* with this it only means that current in conductor XY has no magnetic or any other effect on CD in set{AB-CD} system which is in fact a subset of set{AB-CD-XY} system and is equivalent to case 1.

 

Moreover, saying that there isn’t a potential developed in one and on the other there is like saying -even though the laws of physics are the same for all inertial reference frames, we can have different laws of physics (for the same phenomenon) in the same inertial reference frame.

 

 

 

In another thought experiment, consider a straight long wire, a portion of which is wound around a freely rotating pulley-wheel (so that this arrangement can move along the length of the wire –rotating, winding on one side and unwinding at the other as it is dragged) and this in turn forms the coil of a galvanometer which can deflect a magnetic needle). Fig b.

 

 

movinggalvanometer.png

 

 

Now consider, a long current carrying wire MN which is connected to two such galvanometers G and H in series and these can move along the length of the wire as shown in the fig b. consider two observers GO and HO each with a watch. GO is at rest with respect to the conductor MN and G . Observer HO and H is moving along with the conducting electrons in MN.

So, when asked to measure their time, GO will look at his watch and denote his time. Similarly, HO too will look at his watch which his moving along with him and tells the time which of course agrees with relativity.

 

Now if GO is asked to measure the current and find the cause of the magnetic field, GO will observe it rightfully with the help of G which is in his frame of reference and deciphers that the magnetic field is caused by the flow of electrons.

However if this question is asked to HO, HO instead of looking at H, HO will look at G measures the current and says that magnetic field is caused by the flow of electrons. It is equivalent to HO looking at G’s watch and calling out G’s time.

 

Instead, shouldn’t it be like this- HO should look at H, and measure the current (which is of course has the same magnitude as in G but because of a different reason). H Should notice that the current is caused by the flow of net positive charge and decipher that this is the cause of the magnetic field. Since for H and HO, conducting electrons are at rest and it will not cause a time-variance in electric field. And hence no magnetic field is caused by those electrons. After all we all know that it only charge of the nucleus (just like electronic charge) that we should be concerned with in these scenarios. Why should we give more preference to one of the type of charges over the other just because nucleus makes the bulk of the material?

 

Moreover we often comes to the situation where (if we group the charges of a conductor into two (+) and (-) groups), with respect to an observer, motion of one type of charge group produce a magnetic field while the motion of the other type of charge group doesn’t (and its vice versa where a stationary charge which is not producing any time-variance in its electric field produces a magnetic field). Aren’t these examples for “different laws of physics (for the same phenomenon) in the same inertial reference frame”.

 

(let me stress again that, I do not have any intentions to argue against the theory of relativity. I do believe it’s accurate prediction of various things , from Lorentz contraction to relating energy to mass, to time dilation etc is possible only because it is true and because of this I do believe that Michelson-Morley experiment is interpreted in the right way itself). It’s only against the concept of magnetism that I am arguing with and if this is to change anything regarding relativity, it will be just a few mis-concepts regarding the theory of relativity. The reason for saying that is this, even though theory of relativity wasn’t written with this concept of magnetism in mind, the basic underlying concepts, reasoning and the mathematics associated with it holds true.

 

Thank you Farsight for your reply

 

Farsight in fig3, the current is flowing from positive to neagtive and it is the direction of the current that I have shown in both AB and CD section, ofcouse electrons flow in the opposite direction in AB. If you still see a problem with this do please reply. And regarding the experiment, I just thought of more practical one. either should equally suffice. and there is a bit more in today's post and I dont know how much you will agree with this. Anyway I am waiting for your reply. Thanks

 

 

Pardon me for my possible typing errors in this but do please bring it into my notice so that not only it will help me in eliminating these errors, but also others who are in a way into this post.

Thanking you.

Link to comment
Share on other sites

Thank you Swansont for your reply.

 

what is the cause of the magnetic field in -

 

 

case 1 - where the electrons are at motion with respect to Q

 

and

 

case2 - where the net nuclear positive charge is moving.

 

 

Since the second part of the previous post is not connected to the orginal post

I am re-posting this. sorry if this troubled you.

 

thank you.

Link to comment
Share on other sites

Thank you Swansont for your reply.

 

what is the cause of the magnetic field in -

 

 

case 1 - where the electrons are at motion with respect to Q

 

and

 

case2 - where the net nuclear positive charge is moving.

 

 

Since the second part of the previous post is not connected to the orginal post

I am re-posting this. sorry if this troubled you.

 

thank you.

 

 

The cause of the magnetic field is the moving charge. But that isn't looking at the whole issue. In the case where the wire is moving (the rest frame of the strip) there is also an electric field.

Link to comment
Share on other sites

Thank you Swansont for yor reply and sorry for the delay.

 

is this what you meant with your reply?

 

in case 1 - there is a magnetic field , there is no electric force and there is no magnetic force.

 

and in case 2, where there is a relative velocity between the long wire and CD, there are two cases.

 

 

a) for an observer who is at rest with respect to the wire sees a magnetic force, Fm.

 

and

 

B) for an observer who is moving along with the electrons in the wire, there will be an Electric force , Em such that Em = Fm.

 

my question is

1) in case 1) of course the wire is neutral, but why isnt there any lorentz-contraction taking place, After all the electrons are moving?

 

 

 

thank you.

Edited by abhilash
Link to comment
Share on other sites

You are welcome to try and calculate the effect of a Lorentz-contracted electron, but the force laws already assume point particles. Even if the electron weren't a point particle, the correction would go in the opposite direction — making corrections due to finite size, as we do with atomic physics influences. These are generally small at the atomic level.

Link to comment
Share on other sites

Create an account or sign in to comment

You need to be a member in order to leave a comment

Create an account

Sign up for a new account in our community. It's easy!

Register a new account

Sign in

Already have an account? Sign in here.

Sign In Now
×
×
  • Create New...

Important Information

We have placed cookies on your device to help make this website better. You can adjust your cookie settings, otherwise we'll assume you're okay to continue.